(NIMO 2014) Looks like Cauchy-Schwarz!

Algebra Level 5

For all positive integers k k , define f ( k ) = k 2 + k + 1 f(k)=k^2+k+1 . Compute the largest positive integer n n such that 2015 f ( 1 2 ) f ( 2 2 ) f ( n 2 ) ( f ( 1 ) f ( 2 ) f ( n ) ) 2 . 2015f(1^2)f(2^2)\cdots f(n^2)\geq \Big(f(1)f(2)\cdots f(n)\Big)^2.


The answer is 44.

This section requires Javascript.
You are seeing this because something didn't load right. We suggest you, (a) try refreshing the page, (b) enabling javascript if it is disabled on your browser and, finally, (c) loading the non-javascript version of this page . We're sorry about the hassle.

1 solution

David Altizio
Nov 27, 2015

I claim that f ( n 2 ) = f ( n ) f ( n 1 ) f(n^2)=f(n)f(n-1) for all n 1 n\geq 1 . To prove this, write f ( n 2 ) = n 4 + n 2 + 1 = ( n 2 + 1 ) 2 n 2 = ( n 2 n + 1 ) ( n 2 + n + 1 ) . f(n^2)=n^4+n^2+1=(n^2+1)^2-n^2=(n^2-n+1)(n^2+n+1). Now remark that f ( n 1 ) = ( n 1 ) 2 + ( n 1 ) + 1 = n 2 n + 1 f(n-1)=(n-1)^2+(n-1)+1=n^2-n+1 , so we indeed have f ( n 2 ) = f ( n ) f ( n 1 ) f(n^2)=f(n)f(n-1) as desired.

As a result, dividing through by f ( 1 2 ) f ( 2 2 ) f ( n 2 ) f(1^2)f(2^2)\ldots f(n^2) and engaging in rampant cancellation yields 2015 f ( 1 ) 2 f ( 2 ) 2 f ( n ) 2 f ( 1 2 ) f ( 2 2 ) f ( n 2 ) = f ( 1 ) 2 f ( 2 ) 2 f ( n ) 2 [ f ( 0 ) f ( 1 ) ] [ f ( 1 ) f ( 2 ) ] [ f ( 2 ) f ( 3 ) ] [ f ( n 1 ) f ( n ) ] = f ( 1 ) 2 f ( 2 ) 2 f ( n ) 2 f ( 0 ) f ( 1 ) 2 f ( 2 ) 2 f ( n 1 ) 2 f ( n ) = f ( n ) f ( 0 ) = n 2 + n + 1. \begin{aligned}2015&\geq\dfrac{f(1)^2\cdot f(2)^2\cdot\ldots\cdot f(n)^2}{f(1^2)f(2^2)\ldots f(n^2)}\\&=\dfrac{f(1)^2f(2)^2\ldots f(n)^2}{[f(0)f(1)][f(1)f(2)][f(2)f(3)]\ldots [f(n-1)f(n)]}\\&=\dfrac{f(1)^2f(2)^2\ldots f(n)^2}{f(0)f(1)^2f(2)^2\ldots f(n-1)^2f(n)}\\&=\dfrac{f(n)}{f(0)}=n^2+n+1.\end{aligned} Hence it suffices to find the smallest n n such that 2015 n 2 + n + 1 2015\geq n^2+n+1 . To do this, remark that n 2 + n + 1 n ( n + 1 ) n^2+n+1\approx n(n+1) . From here, one can see that 44 45 = 1980 44\cdot 45=1980 and that 45 46 45\cdot 46 is too large, so n = 44 n=\boxed{44} is the largest value of n n which makes the inequality true.

same method. you could just have used x 1 + 1 + 4 2014 2 x≤\dfrac{-1+\sqrt{1+4*2014}}{2}

Aareyan Manzoor - 5 years, 6 months ago

Exactly Same Way.

Kushagra Sahni - 5 years, 3 months ago

0 pending reports

×

Problem Loading...

Note Loading...

Set Loading...